aide SVP

Aide sur les questions de probabilités.
fatou
Membre
Messages : 12
Inscription : 23 novembre 2013, 22:52

aide SVP

Message par fatou » 27 mars 2015, 20:43

BONJOUR;

aider moi SVP à resoudre cette série
b.jpg
b.jpg (79.74 Kio) Consulté 3454 fois
a.jpg
a.jpg (105.26 Kio) Consulté 3454 fois

Avatar de l’utilisateur
Job
Propriétaire du forum
Messages : 2584
Inscription : 28 juin 2013, 15:07
Contact :

Re: aide SVP

Message par Job » 28 mars 2015, 15:52

Bonjour

Exercice 1
a) On considère un échantillon de 100 personnes.
65 sont des hommes. $65\times \frac{75}{100} = 48,75$ donc 48,75 % de clients masculins sont satisfaits.
35 sont des femmes. $35\times \frac{55}{100}=19,25$ donc 19,25 % de clients féminins satisfaits.
48,75 +19,25=68 donc le taux de satisfaction chez l'ensemble des clients est 68 %

b) $\frac{48,75}{68}\simeq 0,72$ donc la proportion d'hommes par mi les clients satisfaits est de 72 %.

c) Il y a 100 - 68 = 32 clients non satisfaits.
Il y a 35 - 19,25 = 15,75 femmes non satisfaites.
$\frac{15,75}{32}\simeq 0,49$. Parmi les clients non satisfaits il y a 49 % de femmes.

Exercice 2
Tirage sans remise
a) Le nombre de comités possible est : ${12\choose 8}=495$.
Le nombre de comités comprenant les 5 femmes est égal au nombre de choix de 3 hommes parmi 7 soit ${7\choose 3}= 35$
La probabilité que les 5 femmes soient choisies est donc $\frac{35}{495}=\frac{7}{99}$

b) Un comité se compose de 4 hommes pris parmi 7 et d'une femme prise parmi 5 donc ${7\choose 4} \times {5\choose 1}=175$ choix possibles.
Probabilité : $\frac{175}{495} =\frac{35}{99}$.

Tirage avec remise
Cela n'a pas grand sens car une même personne peut être tirée plusieurs fois et le comité n'aura pas ses 8 membres.
La probabilité qu'une femme soit tirée est $\frac{5}{12}$ et un homme : $\frac{7}{12}$
Une éventualité est une suite de 8 éléments H ou F.
Exemple : (F,H,H,F,H,H,H,F) est une éventualité où 3 femmes ont été choisies.
La probabilité de cette éventualité est $ (\frac{5}{12})^3\times (\frac{7}{12})^5=\frac{2100875}{429981696}$
Le nombre de dispositions possibles de 3 femmes dans cette suite de 8 est ${8\choose 3}=35$
Donc la probabilité de tirer 3 femmes est $35\times \frac{2100875}{429981696}=\frac{73530625}{429981696}\simeq 0,17$

Même raisonnement pour avoir 2 hommes :
${8\choose 2} \times (\frac{7}{12})^2 \times (\frac{5}{12})^6\simeq 0,05$

Exercice 5
1. Le nombre de comités possibles est 495. (Voir exercice 2)
Puisqu'il n'y a que 7 hommes, $X$ peut prendre les valeurs de 1 à 5.
En utilisant l'exercice précédent : $P(X=5)=\frac{35}{495}$
$P(X=4)=\frac{{5\choose 4} \times {7\choose 4}}{495}=\frac{5\times 35}{495}=\frac{175}{495}$
$P(X=3)=\frac{{5\choose 3} \times {7\choose 5}}{495} =\frac{10\times 21}{495}=\frac{210}{495}$
$P(X=2)=\frac{{5\choose 2}\times {7\choose 6}}{495} =\frac{10\times 7}{495} =\frac{70}{495}$
$P(X=1)=\frac{{5\choose 1} \times {7\choose 7}}{495} =\frac{5\times 1}{495} =\frac{5}{495}$
(On peut vérifier que la somme des probabilités est bien égale à 1)

2. $E(X)=5\times \frac{35}{495}+4\times \frac{175}{495}+3\times \frac{210}{495}+2\times \frac{70}{495}+1\times \frac{5}{495}=\frac{1650}{495}=\frac{10}{3}$
$E(X^2)=25\times \frac{35}{495}+16\times \frac{175}{495}+9\times \frac{210}{495}+4\times \frac{70}{495}+1\times \frac{5}{495}=\frac{5850}{495}=\frac{130}{11}$
$Var (X)=E(X^2)-(E(X))^2=\frac{130}{11}-\frac{100}{9}=\frac{70}{99}$
$\sigma(X)=\sqrt{\frac{70}{99}}\simeq 0,84$
Calculs à vérifier

Avatar de l’utilisateur
Job
Propriétaire du forum
Messages : 2584
Inscription : 28 juin 2013, 15:07
Contact :

Re: aide SVP

Message par Job » 28 mars 2015, 16:27

Exercice 4
a) $P(0\leq X\leq 0,5)=F(0,5)-F(0)=0,5$

b) La fonction de densité est la dérivée de la fonction de répartition donc $f(x)=\left\{\begin{array}{rcl}0&si&x\leq 0\\1&si&0<x<\leq 1\\ 0&si& x>1\end{array}\right.$

c) $E(X)=\int_0^1 xf(x) dx$ (car en dehors de l'intervalle [0,1], la densité est nulle.
$E(X)=\int_0^1 xdx=[\frac{1}{2}x^2]_0^1=\frac{1}{2}$

$E(X^2)=\int_0^1 x^2 f(x) dx =\int_0^1 x^2 dx =[\frac{1}{3} x^3]_0^1 =\frac{1}{3}$
$Var(X)=E(X^2)-((E(X))^2=\frac{1}{3}-\frac{1}{4} =\frac{1}{12}$
$\sigma(X)=\sqrt{\frac{1}{12}}=\frac{1}{2\sqrt 3}\simeq 0,29$

Exercice 3
a) On doit avoir $\int_2^8 kx^2 dx =1$ soit $[\frac{1}{3} kx^3]_2^8=1$
$\frac{1}{3} k(512-8)=1$ donc $k=\frac{1}{168}$

b) $E(X)=\int_2^8 xkx^2 dx=k\int_2^8 x^3 dx =k[\frac{1}{4} x^4]_2^8=\frac{1}{4\times 468} (8^4-2^4)=\frac{4080}{1872}=\frac{85}{39}$
$E(X^2)=\int_2^8 x^2kx^2 dx =k\int_2^8 x^4dx =k[\frac{1}{5}x^5]_2^8=\frac{1}{5\times 468}(8^5-2^5)=\frac{32736}{2340}=\frac{2728}{195}$
$Var(X)=E(X^2)-(E(X))^2\simeq 9,5$

c) $P(3\leq X\leq 6) =\int_3^6 kx^2 dx =\frac{1}{168} [\frac{1}{3} x^3]_3^6=\frac{1}{504}(6^3-3^3)=\frac{189}{504}=\frac{3}{8}$

fatou
Membre
Messages : 12
Inscription : 23 novembre 2013, 22:52

Re: aide SVP

Message par fatou » 28 mars 2015, 20:39

Merci beaucoup pour votre aide
je vérifierai les calculs

Répondre